Sie sind auf Seite 1von 30

MAGNETISM

20

Responses to Questions
1.

The Earths magnetic field is not always parallel to the surface of the Earthit may have a component
perpendicular to the Earths surface. The compass will tend to line up with the local direction of the
magnetic field, so one end of the compass will dip downward. The angle that the Earths magnetic
field makes with the horizontal is called the dip angle.

2.

The pole on a magnetic compass needle that points geographically northward is defined at the north
pole of the compass. This north pole is magnetically attracted to the south pole of other magnets, so the
Earths magnetic field must have a south pole at the geographic north pole.

3.

The magnetic field lines form clockwise circles centered on the wire.

4.

The force is downward. The field lines point from the north pole to the
south pole, or left to right. Use the right-hand rule. Your fingers point in
the direction of the current (away from you). Curl them in the direction of
the field (to the right). Your thumb points in the direction of the force
(downward). See Fig. 2011a, copied here.

5.

A magnet will not attract just any metallic object. For example, while a magnet will attract paper clips
and nails, it will not attract coins or pieces of aluminum foil. This is because magnets will only attract
other ferromagnetic materials (iron, cobalt, nickel, gadolinium, and some of their oxides and alloys).
Iron and its alloys (such as steel) are the only common materials. These ferromagnetic materials
contain magnetic domains that can be made to temporarily align when a strong magnet is brought near.
The alignment occurs in such a way that the north pole of the domain points toward the south pole of
the strong magnet, and vice versa, which creates the attraction.

6.

No, they are not both magnets. If they were both magnets, then they would repel one another when
they were placed with like poles facing each other. However, if one is a magnet and the other isnt,
then they will attract each other no matter which ends are placed together. The magnet will cause an
alignment of the domains of the nonmagnet, causing an attraction.

Copyright 2014 Pearson Education, Inc. All rights reserved. This material is protected under all copyright laws as they currently exist.
No portion of this material may be reproduced, in any form or by any means, without permission in writing from the publisher.

20-1

20-2

Chapter 20

7.

Typical current in a house circuit is 60 Hz AC. Due to the mass of the compass needle, its reaction to
60 Hz (changing direction back and forth at 60 complete cycles per second) will probably not be
noticeable. A DC current in a single wire could affect a compass, depending on the relative orientation
of the wire and the compass, the magnitude of the current, and the distance from the wire to the
compass. A DC current being carried by two very close wires in opposite directions would not have
much of an effect on the compass needle, since the two currents would cause magnetic fields that
tended to cancel each other.

8.

The magnetic force will be exactly perpendicular to the velocity, which means that the force is
perpendicular to the direction of motion. Since there is no component of force in the direction of
motion, the work done by the magnetic force will be zero, and the kinetic energy of the particle will
not change. The particle will change direction, but not change speed.

9.

Use the right-hand rule to determine the direction of the force on each particle. In the plane of the
diagram, the magnetic field is coming out of the page for points above the wire and is going into the
page for points below the wire.
a: force down, toward the wire
b: force to the left, opposite of the direction of the current
c: force up, toward the wire
d: force to the left, opposite of the direction of the current

10.

Charge a experiences an initial upward force. By the right-hand rule, charge a must be positive.
Charge b experiences no force, so charge b must be uncharged.
Charge c experiences an initial downward force. By the right-hand rule, charge c must be negative.

11.

A magnet can attract an iron bar, so by Newtons third law, an iron bar can attract a magnet. Another
consideration is that the iron has domains that can be made slightly magnetic by an external magnetic
field, as opposed to a substance like plastic or wood. Thus the iron is also a magnet, at least when it is
close to the magnet.

12.

The right-hand rule tells us that as the positive particle moves to the right, if it is deflected upward,
then the magnetic field must be into the page, and if it is deflected downward, then the magnetic field
must be out of the page. Also, the stronger the magnetic field, the more tightly the charged particle will
turn. The magnetic field at the first bend must be relatively small and pointed into the page. The
magnetic field at the second bend must be medium in size and pointed out of the page. The magnetic
field at the third bend must be relatively very large and pointed into the page. In terms of the indicated
letters:
a: no field
b: relatively weak field, into the page
c: moderate strength field, out of the page
d: no field
e: very strong field, into the page

13.

Section 1711 shows that a CRT television picture is created by shooting a beam of electrons at a
fluorescent screen. Wherever the electrons hit the screen, the screen glows momentarily. When you
hold a strong magnet too close to the screen, the magnetic field puts a force on the moving electrons
(F = q B sin ), which causes them to bend away from their original destination position on the
screen. These errant electrons then cause parts of the screen to light up that arent supposed to be lit
and cause other parts to be slightly darker than they are supposed to be. The picture sometimes goes
completely black where the field is strongest because at these points all of the electrons that were
supposed to hit that particular spot have been deflected by the strong field.

Copyright 2014 Pearson Education, Inc. All rights reserved. This material is protected under all copyright laws as they currently exist.
No portion of this material may be reproduced, in any form or by any means, without permission in writing from the publisher.

Magnetism

20-3

14.

Put one end of one rod close to one end of another rod. The ends will either attract or repel. Continue
trying all combinations of rods and ends until two ends repel each other. Then the two rods used in that
case are the magnets.

15.

No, you cannot set a resting electron into motion with a magnetic field (no matter how big the field is).
A magnetic field can only put a force on a moving charge. Thus, with no force (which means no
acceleration), the velocity of the electron will not changeit will remain at rest. However, you can set a
resting electron into motion with an electric field. An electric field will put a force on any charged particle,
moving or not. Thus, the electric force can cause the electron to accelerate from rest to a higher speed.

16.

The particle will move in an elongating helical path in the direction of the electric field (for a positive
charge). The radius of the helix will remain constant, but the pitch (see Problem 79) will increase,
because the particle will accelerate along the electric field lines.

17.

Yes. One possible situation is that the magnetic field is parallel or antiparallel to the velocity of the
charged particle. In this case, the magnetic force would be zero, and the particle would continue moving
in a straight line. Another possible situation is that there is an electric field with a magnitude and direction
(perpendicular to the magnetic field) such that the electric and magnetic forces on the particle cancel each
other out. The net force would be zero and the particle would continue moving in a straight line.

18.

No. A moving charged particle can be deflected sideways with an electric field that points
perpendicular to the direction of the velocity, even when the magnetic field in the region is zero.

19.

Consider that the two wires are both horizontal, with the lower one carrying a current pointing east and
with the upper one carrying a current pointing north. The magnetic field from the upper wire points
downward on the east half of the lower wire and points upward on the west half of the lower wire.
Using the right-hand rule on the east half of the lower wire, where the magnetic field is downward, the
force points to the north. Using the right-hand rule on the west half of the lower wire, where the
magnetic field is upward, the force points to the south. Thus, the lower wire experiences a
counterclockwise torque about the vertical direction due to the magnetic forces from the upper wire.
This torque is attempting to rotate the two wires so that their currents are parallel. Likewise, the upper
wire would experience a clockwise torque tending to align the currents in the wires.

20.

(a)

(b)

21.

The current in the lower wire is pointing in the opposite direction of the current in the upper
wire. If the current in the lower wire is toward the north, then it creates a magnetic field pointing
east at the upper wire. Using the right-hand rule, the current of the upper wire must be pointing
toward the south so that the magnetic force created on it is upward.
No, the lower wire is not stable. If the lower wire falls away from the upper wire, then the
upward magnetic force on it will weaken, and there will be a net downward force on the wire. It
would continue to fall away. Also, if the lower wire were to be moved toward the upper wire, the
upward magnetic force on it would increase, and the lower wire would accelerate upward.

The equation for the magnetic field strength inside a solenoid is given by B =
(a)
(b)
(c)

0 NI

, Eq. 208.
A
The magnetic field strength is not affected if the diameter of the loops doubles. The equation
shows that the magnetic field is independent of the diameter of the solenoid.

If the spacing between the loops doubles, then the length of the solenoid would increase by a
factor of 2, so the magnetic field strength would also decrease by a factor of 2.
If the solenoids length is doubled along with the doubling of the total number of loops, then the
number of loops per unit length remains the same, and the magnetic field strength is not affected.

Copyright 2014 Pearson Education, Inc. All rights reserved. This material is protected under all copyright laws as they currently exist.
No portion of this material may be reproduced, in any form or by any means, without permission in writing from the publisher.

20-4

Chapter 20

22.

To design a relay, place the iron


Springy
Car battery
rod inside of a solenoid and then
Pivot
Solenoid
point one end of the solenoid/rod
Piece
Large current
of
combination at the piece of iron on Iron
Rod
circuit
Iron
a pivot. A spring normally holds
the piece of iron away from a
switch, making an open circuit
Small current
Starter
where current cannot flow. When
circuit for
the relay is activated with a small
dashboard switch
current, a relatively strong
magnetic field is created inside the solenoid, which aligns most of the magnetic domains in the iron
rod and produces a strong magnetic field at the end of the solenoid/rod combination. This magnetic
field attracts the piece of iron on the springy pivot, which causes it to move toward the switch,
connecting it and allowing current to flow through the large current circuit.

23.

The two ions will come out of the velocity selector portion of the mass spectrometer at the same speed,
since = E /B. Once the charges reach the area of the mass spectrometer where there is only a
magnetic field, the difference in the ions charges will have an effect. Eq. 2012 for the mass
spectrometer says that m = qBB r /E , which can be rearranged as r = mE /qBB. Since every quantity
in the equation is constant except q, the doubly ionized ion will hit the film at a half of the radius as the
singly ionized ion.

24.

The magnetic domains in the unmagnetized piece of iron are initially pointing in random directions, as
in Fig. 2042a (which is why it appears to be unmagnetized). When the south pole of a strong external
magnet is brought close to the random magnetic domains of the iron, many of the domains will rotate
slightly so that their north poles are closer to the external south pole, which causes the unmagnetized
iron to be attracted to the magnet. Similarly, when the north pole of a strong external magnet is
brought close to the random magnetic domains of the iron, many of the domains will rotate slightly so
that their south poles are closer to the external north pole, which causes the unmagnetized iron to now
be attracted to the magnet. Thus, either pole of a magnet will attract an unmagnetized piece of iron.

25.

Initially, both the nail and the paper clip have all of their magnetic domains pointing in random
directions (which is why they appear to be unmagnetized). Thus, when you bring them close to each
other, they are not attracted to or repelled from each other. Once the nail is in contact with a magnet
(lets say the north pole), many of the nails domains will align in such a way that the end of the nail
that is touching the magnet becomes a south pole, due to the strong attraction, and the opposite end of
the nail then becomes a north pole. Now, when you bring the nail close to the paper clip, there are
mainly north poles of nail domains close to the paper clip, which causes some of the domains in the
paper clip to align in such a way that the end near the nail becomes a south pole. Since the nails
domains are only partially aligned, it will not be a strong magnet and thus the alignment of the paper
clips domains will be even weaker. The attraction of the paper clip to the nail will be weaker than the
attraction of the nail to the magnet.

Responsess to MisConceptual Questions


1.

(a, b, d, e)
A common misconception is that only permanent magnets (such as a magnet and the
Earth) create magnetic fields. However, moving charges and electric currents also produce
magnetic fields. Stationary charges, ordinary pieces of iron, and other pieces of metal do not
create magnetic fields.

Copyright 2014 Pearson Education, Inc. All rights reserved. This material is protected under all copyright laws as they currently exist.
No portion of this material may be reproduced, in any form or by any means, without permission in writing from the publisher.

Magnetism

20-5

2.

(c)

It is common to confuse the directions of the magnetic fields with electric fields and thus
indicate that the magnetic field points toward or away from the current. Since the current is
flowing into the page, the right-hand rule indicates that the magnetic field is in the clockwise
direction around the current. At point A the field points downward.

3.

(b)

The right-hand rule indicates that the magnetic field is clockwise around the current, so at point
B the current is to the left.

4.

(a)

The charged particle only experiences a force when it has a component of velocity perpendicular
to the magnetic field. When it moves parallel to the field, it follows a straight line at constant
speed.

5.

(c)

Electric fields are created by charged objects whether the charges are moving or not. Magnetic
fields are created by moving charged objects. Since the proton is charged and moving, it creates
both an electric field and a magnetic field.

6.

(a)

A stationary charged particle does not experience a force in a magnetic field. Therefore, the
particle must be moving to experience a force. The force is a maximum when the particle is
moving perpendicular to the field, not parallel to the field. Since the force is perpendicular to the
motion of the particle, it acts as a centripetal force, changing the particles direction but not its
kinetic energy. That is, since the force is perpendicular to the motion, it does no work on the
particle. The direction of the force is always perpendicular to the direction of motion and also
perpendicular to the magnetic field.

7.

(c)

Section 205 shows that the magnetic field from a current is directed circularly around the wire,
is proportional to the current flowing in the wire, and is inversely proportional to the distance
from the wire. A constant current produces a magnetic field, so the current does not need to be
changing.

8.

(e)

It is common to confuse the direction of electric fields (which point toward or away from the
charges) with magnetic fields, which always make circles around the current.

9.

(c)

A common misconception is that a force always does work on the object. Since the magnetic
force is perpendicular to the velocity of the proton, the force acts as a centripetal force, changing
the protons direction, but not doing any work and thus not changing its kinetic energy.

10.

(a)

A common misconception is that a constant magnetic field can change the magnitude of the
particles velocity. However, the magnetic force is always perpendicular to the velocity, so it can
do no work on the particle. The magnetic force only serves as a centripetal force to change the
particles direction.

11.

(e)

Equation 203 shows that the magnetic force depends upon the particles charge, its velocity,
and the strength of the external magnetic field. The direction of the force is always perpendicular
to the magnetic field and the velocity of the particle. Therefore, all four statements are accurate.

12.

(c)

This question requires a consideration of Newtons third law. The force that one wire exerts on a
second must be equal in magnitude, but opposite in direction, to the force that the second exerts
on the first.

Copyright 2014 Pearson Education, Inc. All rights reserved. This material is protected under all copyright laws as they currently exist.
No portion of this material may be reproduced, in any form or by any means, without permission in writing from the publisher.

20-6

Chapter 20

Solutions to Problems
1.

(a)

Use Eq. 201 to calculate the force with an angle of 90 and a length of 1 meter.
F = I AB sin

(b)

F /A = IB sin = (6.40 A)(0.90 T) sin 90 = 5.8 N/m

Change the angle to 35.0.


F /A = IB sin = (6.40 A)(0.90 T) sin 35.0 = 3.3 N/m

2.

Use Eq. 202.


Fmax = I AB I =

3.

Fmax
0.625 N
=
= 1.63 A
AB
(4.80 m)(8.00 102 T)

Use Eq. 201 to calculate the force.


F = I AB sin = (120 A)(240 m)(5.0 105 T) sin 68 = 1.3 N

4.

The dip angle is the angle between the Earths magnetic field and the current in the wire. Use Eq. 201
to calculate the force.
F = I AB sin = (4.5 A)(2.6 m)(5.5 105 T) sin 41 = 4.2 104 N

5.

To have the maximum force, the current must be perpendicular to the magnetic field,
F
F
= 0.45 max to find the angle between the wire
as shown in the first diagram. Use
A
A
and the magnetic field, illustrated in the second diagram. Use Eq. 201.
F
F
= 0.45 max
A
A

6.

IB sin = 0.45IB = sin

0.45 = 27

wire

G
B

wire

Use Eq. 202. The length of wire in the B field is the same as the diameter of the pole faces.
Fmax = I AB B =

7.

G
B

Fmax
1.28 N
=
= 0.358 T
IA
(6.45 A)(0.555 m)

(a)

By the right-hand rule, the magnetic field must be pointing up, so the top pole face must be a
south pole.

(b)

Use Eq. 202 to relate the maximum force to the current. The length of wire in the magnetic
field is equal to the diameter of the pole faces.
Fmax = I AB I =

(c)

Fmax
(8.50 102 N)
=
= 3.864 A 3.86 A
AB
(0.100 m)(0.220 T)

If the wire is tipped so that it points 10.0 downward, then


the angle between the wire and the magnetic field is
changed to 100.0. But the length of wire now in the field
has increased from A to A / cos10.0. The net effect of the
changes is that the force has not changed.

initial
rotated

Finitial = I AB; Frotated = I (A / cos10.0) B sin100.0 = I AB = Finitial = 8.50 102 N


Copyright 2014 Pearson Education, Inc. All rights reserved. This material is protected under all copyright laws as they currently exist.
No portion of this material may be reproduced, in any form or by any means, without permission in writing from the publisher.

Magnetism

8.

20-7

The magnetic force must be equal in magnitude to the force of gravity on the wire. The maximum
magnetic force is applicable since the wire is perpendicular to the magnetic field. The mass of the wire
is the density of copper times the volume of the wire.
FB = mg
I=

I AB =

d 2 g
4B

( 12 d )

Ag

(8.9 103 kg/m3 ) (1.00 103 m) 2 (9.80 m/s 2 )


4(5.0 105 T)

= 1400 A

This answer does not seem feasible. The current is very large, and the resistive heating in the thin
copper wire would probably melt it.
9.

The maximum magnetic force as given in Eq. 204 can be used since the velocity is perpendicular to
the magnetic field.
Fmax = q B = (1.60 1019 C)(7.75 105 m/s)(0.45 T) = 5.6 1014 N
By the right-hand rule, the force must be directed to the north.

10.

The magnetic force will cause centripetal motion, and the electron will move in a clockwise circular
path if viewed in the direction of the magnetic field. The radius of the motion can be determined.
Fmax = q B = m
r=

2
r

m (9.11 1031 kg)(1.70 106 m/s)


=
= 1.51 105 m
qB
(1.60 1019 C)(0.640 T)

Assuming the magnetic field existed in a sharply defined region, the electron would make only onehalf of a rotation and then exit the field going in the opposite direction from which it came.
11.

In this scenario, the magnetic force is causing centripetal motion, so it must have the form of a
centripetal force. The magnetic force is perpendicular to the velocity at all times for circular motion.
Fmax = q B = m

2
r

B=

m (6.6 1027 kg)(1.6 106 m/s)


=
= 0.24 T
qr
2(1.60 1019 C)(0.14 m)

12.

Since the charge is negative, the answer is the OPPOSITE of the result given from the right-hand rule
applied to the velocity and magnetic field.
(a) no force
(b) downward
(c) upward
(d) inward, into the page
(e) to the left
(f) to the left

13.

The right-hand rule applied to the velocity and magnetic field would give the direction of the force.
Use this to determine the direction of the magnetic field given the velocity and the force.
(a) to the right
(b) downward
(c) into the page

Copyright 2014 Pearson Education, Inc. All rights reserved. This material is protected under all copyright laws as they currently exist.
No portion of this material may be reproduced, in any form or by any means, without permission in writing from the publisher.

20-8

14.

Chapter 20

The force on the electron due to the electric force must be the same magnitude as the force on the
electron due to the magnetic force.
FE = FB

qE = q B =

E 7.7 103 V/m


=
= 1.027 106 m/s 1.0 106 m/s
B
7.5 103 T

If the electric field is turned off, then the magnetic force will cause circular motion.
FB = q B = m

15.

(a)

r=

The speed of the ion can be found using energy conservation. The electric potential energy of the
ion becomes kinetic energy as it is accelerated.
qV = 12 m 2

Einitial = Efinal

=
(b)

(6.6 1027 kg)

2
r

m (6.6 1027 kg)(5.99 105 m/s)


=
= 3.63 102 m 3.6 102 m
qB
2(1.60 1019 C)(0.340 T)

The period can be found from the speed and the radius.
T=

(a)

= 5.99 105 m/s 6.0 105 m/s

Since the ion is moving perpendicular to the magnetic field, the magnetic force will be a
maximum. That force will cause the ion to move in a circular path.

r=

16.

2[2(1.60 1019 C)](3700 V)

2qV
=
m

Fmax = q B = m

(c)

m (9.11 1031 kg)(1.027 106 m/s)


=
= 7.8 104 m
qB
(1.60 1019 C)(7.5 103 T)

2 r

2 (3.63 102 m)
5.99 105 m/s

= 3.8 107 s

From Example 206, we have r =

rqB
m
. The kinetic energy is given by
, so =
m
qB

r 2q2 B2
rqB
= 12 m 2 = 12 m
, so we see that KE r 2 .
=
2m
m

KE

(b)

The angular momentum of a particle moving in a circular path is L = m r. From Example 206,
rqB
m
we have r =
, so =
. Combining these relationships gives the following.
m
qB
L = m r = m

17.

rqB
r = qBr 2
m

The kinetic energy of the proton can be used to find its velocity. The magnetic force produces
centripetal acceleration, and from this the radius can be determined.
KE

= 12 m 2

m
r=
=
qB

=
2 KE
m =
qB

2 KE
m
2 KE m
=
qB

q B =

m 2
r

2(1.5 106 eV)(1.60 1019 J/eV)(1.67 1027 kg)


(1.60 1019 C)(0.30 T)

= 0.59 m

Copyright 2014 Pearson Education, Inc. All rights reserved. This material is protected under all copyright laws as they currently exist.
No portion of this material may be reproduced, in any form or by any means, without permission in writing from the publisher.

Magnetism

18.

20-9

The magnetic field can be found from Eq. 204, and the direction is found from the right-hand rule.
Remember that the charge is negative.
Fmax = q B B =

Fmax
6.2 1013 N
=
= 1.4 T
q
(1.60 1019 C)(2.8 106 m/s)

The direction would have to be east for the right-hand rule, applied to the velocity and the magnetic
field, to give the proper direction of force.
19.

The kinetic energy is used to determine the speed of the particles, and then the speed can be used to
determine the radius of the circular path, since the magnetic force is causing centripetal acceleration.

KE

= 12 m 2
2mp

rp
re

20.

m
r=
=
qB

2 KE
m =
qB

2m KE
qB

KE

qB
2me KE
qB

mp

me

1.67 1027 kg

9.11 1031 kg

= 42.8

The velocity of each charged particle can be found using energy conservation. The electrical potential
energy of the particle becomes kinetic energy as it is accelerated. Then, since the particle is moving
perpendicularly to the magnetic field, the magnetic force will be a maximum. That force will cause the
ion to move in a circular path, and the radius can be determined in terms of the mass and charge of the
particle.
Einitial = Efinal

Fmax = q B = m

qV = 12 m 2

1 2mdV
B
qd

rd
=
=
rp 1 2mpV
B
qp
1 2m V
B
q

r
=
=
rp
1 2mpV
B
qp

21.

m 2
q B =
r

2 KE
m

r=
md
mp
qd
qp
m
mp
q
qp

2
1

4
2

m
=
qB

= 2

= 2

2qV
m

2qV
m = 1 2mV
qB
B
q

rd = 2rp

r = 2rp

The magnetic force produces an acceleration perpendicular to the original motion. If that acceleration
is small, then it will produce a small deflection, and the original velocity can be assumed to always be
perpendicular to the magnetic field. This leads to a constant perpendicular acceleration. The time that
this (approximately) constant acceleration acts can be found from the original velocity and the
distance traveled A. The starting speed in the perpendicular direction will be zero.

Copyright 2014 Pearson Education, Inc. All rights reserved. This material is protected under all copyright laws as they currently exist.
No portion of this material may be reproduced, in any form or by any means, without permission in writing from the publisher.

20-10

Chapter 20

F = ma = q B a =
d = 0 t + 12 a t 2 =

q B
m
2

1
2

q B A
qBA 2 (18.5 109 C)(5.00 105 T)(1.50 103 m) 2
=
=

m
2m
2(3.40 103 kg)(155 m/s)

= 1.97 106 m

This small distance justifies the assumption of constant acceleration.


22.

(a)

The discussion of the Hall effect in the text states that the Hall emf is proportional to the
magnetic field causing the effect. We can use this proportionality to determine the unknown
resistance. Since the new magnetic field is oriented 90 to the surface, the full magnetic field will
be used to create the Hall potential.
VHall B
=
VHall B

(b)

VHall
B
VHall

B =

63 mV (0.10 T)
= 0.61 T
12 mV sin 60

(a)

The sign of the ions will not change the magnitude of the Hall emf but will determine the
polarity of the emf.

(b)

The flow velocity corresponds to the drift velocity in the Hall effect relationship.
VHall = Bd

24.

VHall
63 mV
B =
(0.10 T) = 0.53 T
VHall
12 mV

When the field is oriented at 60 to the surface, the magnetic field, B sin 60, is used to create
the Hall potential.
B sin 60 =

23.

B =

(a)

VHall
(0.13 103 V)
=
= 0.56 m/s
Bd
(0.070 T)(0.0033 m)

The drift velocity is found from VHall = d Bd .

d =
(b)

VHall
(1.02 106 V)
=
= 4.722 105 m/s 4.7 105 m/s
Bd
(1.2 T)(0.018 m)

We now find the density by using Eq. 1810.


n=

I
15 A
=

19

3
eAd (1.6 10
C)(1.0 10 m)(0.018 m)(4.722 105 m/s)

= 1.1 1029 electrons/m3

25.

We assume that the jumper cable is a long straight wire and use Eq. 206.
Bcable =

0 I (4 107 T m/A)(65 A)
=
= 2.889 104 T 2.9 104 T
2 r
2 (4.5 102 m)

Compare this with the Earths field of 0.5 104 T.


Bcable /BEarth =

2.889 104 T
5.0 105 T

= 5.77, so the field of the cable is about 5.8 times that of the Earth.

Copyright 2014 Pearson Education, Inc. All rights reserved. This material is protected under all copyright laws as they currently exist.
No portion of this material may be reproduced, in any form or by any means, without permission in writing from the publisher.

Magnetism

26.

We assume that the wire is long and straight and use Eq. 206.
Bwire =

27.

0 I
2 r

I=

2 rBwire

2 (0.12 m)(0.50 104 T)


4 107 T m/A

= 30 A (2 significant figures)

Since the currents are parallel, the force on each wire will be attractive, toward the other wire. Use
Eq. 207 to calculate the magnitude of the force.
F2 =

28.

20-11

0 I1 I 2
(4 107 T m/A) (25 A) 2
(25 m) = 7.8 102 N, attractive
A2 =
2 d
2
(0.040 m)

Since the force is attractive, the currents must be in the same direction, so the current in the second
wire must also be upward. Use Eq. 207 to calculate the magnitude of the second current.

0 I1 I 2
A2
2 d
2 F2 d
2
0.090 m
I2 =
=
= 12.54 A 13 A upward
(7.8 104 N/m)
7

0 A 2 I1 4 10 T m/A
28 A

F2 =

29.

To find the direction, draw a radius line from the wire to the field
point. Then at the field point, draw a line perpendicular to the radius,
directed so that the perpendicular line would be part of a
counterclockwise circle. The relative magnitude is given by the
length of the arrow. The farther a point is from the wire, the weaker
the field.

30.

For the experiment to be accurate to 3.0%, the magnetic field due to the current in the cable must be
less than or equal to 3.0% of the Earths magnetic field. Use Eq. 206 to calculate the magnetic field
due to the current in the cable.
Bcable =

0 I
0.030 BEarth
2 r

2 r (0.030 BEarth )

2 (1.00 m)(0.030)(0.50 104 T)


4 107 T m/A

= 7.5 A

Thus the maximum allowable current is 7.5 A .


31.

The magnetic field at the loop due to the long wire is into the page and can be calculated by Eq. 206.
The force on the segment of the loop closest to the wire is toward the wire, since the currents are in the
same direction. The force on the segment of the loop farthest from the wire is away from the wire,
since the currents are in the opposite direction.
Because the magnetic field varies with distance, it is difficult to calculate the total force on either the
left or right segments of the loop. Using the right-hand rule, the force on each small piece of the left
segment of wire is to the left, and the force on each small piece of the right segment of wire is to the
right. If left and right small pieces are chosen that are equidistant from the long wire, then the net force
on those two small pieces is zero. Thus the total force on the left and right segments of wire is zero, so
only the parallel segments need to be considered in the calculation. Use Eq. 207 to find the force.
Fnet = Fnear Ffar =
=

1
0 I1 I 2
II

A near 0 1 2 A far = 0 I1 I 2 A

2 d near
2 d far
2
d near dfar

4 107 T m/A
1
1
6

(3.5 A) 2 (0.100 m)
= 5.1 10 N, toward wire
2
0.030
m
0.080
m

Copyright 2014 Pearson Education, Inc. All rights reserved. This material is protected under all copyright laws as they currently exist.
No portion of this material may be reproduced, in any form or by any means, without permission in writing from the publisher.

20-12

32.

Chapter 20

At the location of the compass, the magnetic field caused by the wire will point
to the west, and the part of the Earths magnetic field that turns the compass is
pointing due north. The compass needle will point in the direction of the NET
magnetic field.
Bwire

I (4 107 T m/A)(48 A)
= 0 =
= 5.33 105 T
2 r
2 (0.18 m)

G
B net

G
B Earth

G
B wire

BEarth
4.5 105 T
= tan 1
= 40 N of W (2 significant figures)
Bwire
5.33 105 T
G
B
wire
The magnetic field due to the long horizontal wire points straight up at
the point in question, and its magnitude is given by Eq. 206. The two
fields are oriented as shown in the diagram. The net field is the vector
sum of the two fields.

= tan 1

33.

Bwire

I (4 10 T m/A)(24.0 A)
= 0 =
= 2.40 105 T
2 r
2 (0.200 m)

44

G
B Earth

G
B net

BEarth = 5.0 105 T


Bnet x = BEarth cos 44 = 3.60 105 T

Bnet, y = Bwire BEarth sin 44 = 1.07 105 T

2
2
5
Bnet = Bnet
T) 2 + (1.07 105 T) 2 = 3.8 105 T
x + Bnet y = (3.60 10

= tan 1
34.

Bnet x

1.07 105 T
3.60 105 T

= 17 below the horizontal

(a)

0 I (4 107 T m/A)(2.5 109 protons/s)(1.60 1019 C/proton)


=
= 5.3 1017 T
2 r
2 (1.5 m)

If the currents are in the same direction, then the magnetic fields at the midpoint between the two
currents will oppose each other, so their magnitudes should be subtracted.
Bnet =

(b)

0 I1 0 I 2 (4 107 T m/A)

=
(I 25 A) = (2.0 105 T/A)(I 25 A)
2 r1 2 r2
2 (0.010 m)

If the currents are in the opposite direction, then the magnetic fields at the midpoint between the
two currents will reinforce each other, so their magnitudes should be added.
Bnet =

36.

= tan 1

The stream of protons constitutes a current, whose magnitude is found by multiplying the proton rate
by the charge of a proton. Then use Eq. 206 to calculate the magnetic field.
Bstream =

35.

Bnet y

0 I1 0 I 2 (4 107 T m/A)
+
=
(I + 25 A) = (2.0 105 T/A)(I + 25 A)
2 r1 2 r2
2 (0.010 m)

Using the right-hand rule, we see that if the currents flow in the same direction, then the magnetic
fields will oppose each other between the wires and therefore can equal zero at a given point. Set the
sum of the magnetic fields from the two wires equal to zero at the point 2.2 cm from the first wire and
use Eq. 206 to solve for the unknown current.
Bnet = 0 =

0 I1 0 I 2

2 r1 2 r2

r
7.0 cm 2.2 cm
I 2 = 2 I1 =
(2.0 A) = 4.4 A, same direction as other current
2.2 cm

r1
Copyright 2014 Pearson Education, Inc. All rights reserved. This material is protected under all copyright laws as they currently exist.
No portion of this material may be reproduced, in any form or by any means, without permission in writing from the publisher.

Magnetism

37.

Use the right-hand rule to determine the direction of the magnetic field from each
wire. Remembering that the magnetic field is inversely proportional to the distance
from the wire, qualitatively add the magnetic field vectors. The magnetic field at
point 2 is zero.

38.

(a)

20-13

We assume that the power line is long and straight and use Eq. 206.
Bline =

0 I (4 107 T m/A)(95 A)
=
= 2.235 106 T 2.2 106 T
2 r
2 (8.5 m)

The direction at the ground, from the right-hand rule, is south. Compare this with the Earths
field of 0.5 104 T, which points approximately north.
Bline /BEarth =

2.235 106 T
0.5 104 T

= 0.0447

The field of the cable is about 4% that of the Earth.


(b)

We solve for the distance where Bline = BEarth .


Bline =

0 I
= BEarth
2 r

r=

0 I

2 BEarth

(4 107 T m/A)(95 A)
2 (0.5 104 T)

= 0.38 m 0.4 m

So about 0.4 m below the wire, the net B field would be 0, assuming that the Earths field points
straight north at this location.
39.

The Earths magnetic field is present at both locations in the problem, and we
assume that it is the same at both locations. The field east of a vertical wire must
be pointing either due north or due south. The compass shows the direction of the
net magnetic field, and it changes from 17 E of N to 32 E of N when taken
inside. That is a southerly change (rather than a northerly change), so the
field due to the wire must be pointing due south. See the diagram, in which
= 17, + = 32, and + + = 180. Thus = 15 and = 148. Use the
G
law of sines to find the magnitude of B wire and then use Eq. 206 to find the
magnitude of the current.
Bwire BEarth
=
sin
sin
I = BEarth

Bwire = BEarth

sin 0 I
=
sin 2 r

G
B Earth

BG

wire

G
B net

sin 2
sin15
2
r = (5.0 105 T)
(0.120 m) = 14.65 A 15 A

sin 0
sin 148 4 10 7 T m/A

Since the field due to the wire is due south, the current in the wire must be downward.
40.

The fields created by the two wires will oppose each other, so the net field is the difference between
the magnitudes of the two fields. The positive direction for the fields is taken to be into the page, so the
closer wire creates a field in the positive direction, and the more distant wire creates a field in the
negative direction. Let d be the separation distance of the wires.
Bnet =

0 I
0 I
I 1
1

= 0

2 rcloser 2 rfarther
2 rcloser rfarther

0 I 1
1

=
1
1
2 r 2 d r + 2 d

Copyright 2014 Pearson Education, Inc. All rights reserved. This material is protected under all copyright laws as they currently exist.
No portion of this material may be reproduced, in any form or by any means, without permission in writing from the publisher.

20-14

Chapter 20

0 I
d

2 ( r 1 d )( r + 1 d )

(4 10

T m/A)(24.5 A)
0.0028 m

2
(0.10 m 0.0014 m)(0.10 m + 0.0014 m)

= 1.372 106 T 1.4 106 T

Compare this with the Earths field of 0.5 104 T.


Bnet /BEarth =

1.372 106 T
0.5 104 T

= 0.027

The field of the wires is about 3% that of the Earth.


41.

The center of the third wire is 5.6 mm from the left wire and 2.8 mm from the right wire. The force on
the near (right) wire will attract the near wire, since the currents are in the same direction. The force
on the far (left) wire will repel the far wire, since the currents oppose each other. Use Eq. 207 to
calculate the force per unit length.
0 I1 I 2

Fnear =
Fnear

A near

A far

42.

2 d near

2 d far

A near

0 I1 I 2

0 I1 I 2

Ffar =
Ffar

2 d near

0 I1 I 2
2 d far

A far
=

4 107 T m/A (25.0 A)(24.5 A)


= 4.4 10 2 N/m, attract (to the right)
3
2
2.8 10 m

4 107 T m/A ( 25.0 A)( 24.5 A)


= 2.2 10 2 N/m, repel (to the left)
2
5.6 103 m

Since the magnetic field from a current-carrying wire circles the


wire, the individual field at point P from each wire is perpendicular
G
to the radial line from that wire to point P. We define B1 as the field
G
from the top wire and B 2 as the field from the bottom wire. We use
Eq. 206 to calculate the magnitude of each individual field.
B1 =

0 I (4 107 T m/A)(28 A)
=
= 9.333 105 T
2 r1
2 (0.060 m)

B2 =

0 I (4 107 T m/A)(28 A)
=
= 5.600 105 T
2 r2
2 (0.100 m)

We use the law of cosines to determine the angle that the radial line
from each wire to point P makes with the vertical. Since the field is perpendicular to the radial line,
this is the same angle that the magnetic fields make with the horizontal.
(0.060 m) 2 + (0.130 m) 2 (0.100 m) 2
= 47.7

2(0.060 m)(0.130 m)

2
2
2
(0.100 m) + (0.130 m) (0.060 m)
2 = cos 1
= 26.3

2(0.100 m)(0.130 m)

1 = cos 1

Copyright 2014 Pearson Education, Inc. All rights reserved. This material is protected under all copyright laws as they currently exist.
No portion of this material may be reproduced, in any form or by any means, without permission in writing from the publisher.

Magnetism

20-15

Using the magnitudes and angles of each magnetic field, we calculate the horizontal and vertical
components, add the vectors, and calculate the resultant magnetic field and angle.
Bnet x = B1 cos (1 ) B2 cos 2 = (9.333 105 T) cos 47.7 (5.600 105 T) cos 26.3 = 1.261 105 T
Bnet y = B1 sin (1 ) + B2 sin 1 = (9.333 104 T) sin 47.7 + (5.600 105 T) sin 26.3 = 9.384 105 T
2
2
5
B = Bnet,
T) 2 + (9.384 105 T) 2 = 9.468 105 T
x + Bnet, y = (1.261 10

= tan 1

Bnet , y
Bnet, y

= tan 1

8.379 105 T
1.126 105 T

= 82.3

G
B = 9.468 105 T at 82.3 9.5 105 T at 82

43.

The magnetic fields created by the individual currents will be at right angles to each other. The field
due to the top wire will be to the right, and the field due to the bottom wire will be out of the page.
Since they are at right angles, the net field is the hypotenuse of the two individual fields.

Bnet

0 I top
=
2 rtop

0 I bottom 2

4 107 T m/A
2
2
= 0 I top
+ I bottom
=
(20.0 A) 2 + (12.0 A) 2
+

2 rbottom
2
r
2
(0.100
m)

= 4.66 105 T

44.

Use Eq. 208 for the field inside a solenoid.


B=

45.

(4 107 T m/A)(460)(2.0 A)
= 9.6 103 T
0.12 m

0 NI
A

I=

BA
(4.65 103 T)(0.300 m)
=
= 1.19 A
0 N (4 107 T m/A)(935)

The field inside a solenoid is given by Eq. 208.


B=

47.

Use Eq. 208 for the field inside a solenoid.


B=

46.

0 NI

0 NI
A

N=

BA
(0.030 T)(0.42 m)
=
= 2.2 103 turns
0 I (4 107 T m/A)(4.5 A)

The field due to the solenoid is given by Eq. 208. Since the field due to the solenoid is perpendicular
to the current in the wire, Eq. 202 can be used to find the force on the wire segment.
F = I wire A wire Bsolenoid = I wire A wire

0 NI solenoid
A solenoid

= (22 A)(0.030 m)

(4 107 T m/A)(38 A)(550)


(0.15 m)

= 0.1156 N 0.12 N to the south

Copyright 2014 Pearson Education, Inc. All rights reserved. This material is protected under all copyright laws as they currently exist.
No portion of this material may be reproduced, in any form or by any means, without permission in writing from the publisher.

20-16

48.

Chapter 20

Since the mass of copper is fixed and the density is fixed, the volume of copper is fixed, and we
designate it as VCu = mCu Cu = A Cu ACu . We call the fixed voltage V0 . The magnetic field in the
solenoid is given by Eq. 208. For the resistance, we used the resistivity-based definition from Eq. 183,
with resistivity represented by R Cu .
B=

0 NI
A sol

= 0

N V0
N
= 0
A sol RCu
A sol

V0

RCu

A Cu
ACu

0V0 N ACu 0V0 N mCu Cu


=
RCu A sol A Cu RCu A sol A 2Cu

0V0 mCu Cu N
R Cu
A sol A 2Cu

The number of turns of wire is the length of wire divided by the circumference of the solenoid.

N=

A Cu
2 rsol

B=

A Cu
2 rsol

0V0 mCu Cu N
Vm
Vm
1
= 0 0 Cu Cu
= 0 0 Cu Cu
2
2
A
2
r
RCu

A sol A Cu
A sol A Cu
R Cu
RCu
sol sol A Cu

The first factor in the expression for B is made of constants, so we have B

. Thus we
A sol rsol A Cu
want the wire to be short and fat. Also, the radius of the solenoid should be small and the length of the
solenoid should be small.

49.

(a)

Each loop of wire produces a field along its axis, similar to Fig. 209. For path 1, with all the
loops taken together, that symmetry leads to a magnetic field that is the same anywhere along the
path and parallel to the path. One side of every turn of the wire is enclosed by path 1, so the
enclosed current is NI . Apply Eq. 209.

B& A = 0 NI
(b)

B = 0 NI /2 R

For path 2, each loop of wire pierces the enclosed area twiceonce going up and once going
down. Amperes law takes the direction of the current into account, so the net current through
the area enclosed by path 2 is 0.

B& A = 0 NI
(c)

B(2 R ) = 0 NI

B(2 R ) = 0

B=0

The field inside a toroid is not uniform. As seen in the result to part (a), the field varies inversely
as the radius of the toroid, so B

1
. The field is strongest at the inside wall of the toroid, and
R

weakest at the outside wall.


50.

(a)

Amperes law (Eq. 209) says that along a closed path,

B& A = 0 I encl . For the path, choose

a circle of radius r, centered on the center of the wire, greater than the radius of the inner wire
and less than the radius of the outer cylindrical braid. Because the wire is long and straight, the
magnetic field is tangent to the chosen path, so B& = B. The current enclosed is I.

0 I encl = B& A = BA = B A = B(2 r ) B =

0 I
2 r

Copyright 2014 Pearson Education, Inc. All rights reserved. This material is protected under all copyright laws as they currently exist.
No portion of this material may be reproduced, in any form or by any means, without permission in writing from the publisher.

Magnetism

(b)

20-17

We make a similar argument, but now choose the path to be a circle of radius r, greater than the
radius of the outer cylindrical braid. Because the wire and the braid are long and straight, the
magnetic field is tangent to the chosen path, so B& = B. The current enclosed by the path is zero,
since there are two equal but oppositely directed currents.

0 I encl = B& A = BA = B A = B(2 r ) B =


51.

0 I encl
=0
2 r

If the face of the loop of wire is parallel to the magnetic field, then the angle between the perpendicular
to the loop and the magnetic field is 90. Use Eq. 2010 to calculate the magnetic field strength.

= NIAB sin

B=

NIA sin

0.325 m N
(1)(5.70 A)(0.220 m)2 sin 90

= 1.18 T

52.

From Eq. 2010, we see that the torque is proportional to the current, so if the current drops by 12%,
then the output torque will also drop by 12%. Thus the final torque is 0.88 times the initial torque.

53.

In Section 2010, it is shown that the angular deflection of the galvanometer needle is proportional to
the product of the current and the magnetic field. Thus if the magnetic field is decreased to 0.760 times
its original value, then the current must be increased by dividing the original value by 0.760 to obtain
the same deflection.
(IB)initial = (IB )final

54.

(a)

I final =

I initial Binitial (53.0 A)Binitial


=
= 69.7 A
Bfinal
0.760 Binitial

The torque is given by Eq. 2010. The angle is the angle between the B field and the
perpendicular to the coil face.
0.120 m 2
5
5
(5.50 10 T) sin 34.0 = 2.25 10 m N
2

= NIAB sin = 9(7.20 A)


(b)

55.

In Example 2013 it is stated that if the coil is free to turn, then it will rotate toward the
orientation so that the angle is 0. In this case, that means the north edge of the coil will rise, so
that a perpendicular to its face will be parallel with the Earths magnetic field.

The radius and magnetic field values can be used to find the speed of the protons. The electric field is
then found from the fact that the magnetic force must be the same magnitude as the electric force for
the protons to have straight paths.
q B = m 2 /r

= qBr /m

E = B = qB 2 r /m =

(1.60 10

19

qE = q B

FE = FB

C)(0.566 T) (6.10 102 m)


2

1.67 1027 kg

= 1.87 106 V/m

The direction of the electric field must be perpendicular to both the velocity and the magnetic field and
must be in the opposite direction to the magnetic force on the protons.

Copyright 2014 Pearson Education, Inc. All rights reserved. This material is protected under all copyright laws as they currently exist.
No portion of this material may be reproduced, in any form or by any means, without permission in writing from the publisher.

20-18

56.

Chapter 20

The magnetic force on the ions causes them to move in a circular path, so the magnetic force is a
centripetal force. This results in the ion mass being proportional to the paths radius of curvature.
q B = m 2 /r m = qBr / m /r = qB / = constant = 76 u/22.8 cm
m21.0
m21.6
76 u
76 u
=
m21.0 = 70 u
=
m21.6 = 72 u
21.0 cm 22.8 cm
21.6 cm 22.8 cm
m21.9
m22.2
76 u
76 u
=
m21.9 = 73 u
=
m22.2 = 74 u
21.9 cm 22.8 cm
22.2 cm 22.8 cm

The other masses are 70 u, 72 u, 73 u, and 74 u.


57.

The location of each line on the film is twice the radius of curvature of the ion. The radius of curvature
can be found from Eq. 2012.
m=

qBB r
E

2r12 =

r=

mE
qBB

2r =

2mE
qBB

2(12)(1.67 1027 kg)(2.88 104 V/m)


(1.60 10

2r13 = 1.690 10

19

C)(0.68 T)

= 1.560 102 m

2r14 = 1.820 102 m

The distances between the lines are the following.


2r13 2r12 = 1.690 102 m 1.560 102 m = 1.11 103 m 1.3 103 m
2r14 2r13 = 1.820 102 m 1.690 102 m = 1.12 103 m 1.3 103 m

If the ions are doubly charged, then the value of q in the denominator of the expression would double,
so the actual distances on the film would be halved. Thus the distances between the lines would also be
halved.
2r13 2r12 = 8.451 103 m 7.801 103 m = 6.5 104 m
2r14 2r13 = 9.101 103 m 8.451 103 m = 6.5 104 m

58.

The velocity of the ions is found using energy conservation. The electrical potential energy of the ions
becomes kinetic energy as they are accelerated. Then, since the ions move perpendicularly to the
magnetic field, the magnetic force will be a maximum. That force will cause the ions to move in a
circular path.
q B =

59.

m 2
R

qBR
m

q2 B2 R2
qBR
qV = 12 m 2 = 12 m
=

2m
m

m=

qR 2 B 2
2V

Since the particle is undeflected in the crossed fields, its speed is given by = E /B, as stated in
Section 2011. Without the electric field, the particle will travel in a circle due to the magnetic force.
Using the centripetal acceleration, we can calculate the mass of the particle. Also, the charge must be
an integer multiple of the fundamental charge.
q B =
m=

m 2
r

qBr

qBr
neB 2 r n(1.60 1019 C)(0.034 T) 2 (0.027 m)
=
=
= n(3.3 1027 kg) n(2.0 u)
E
(E /B )
1.5 103 V/m

Copyright 2014 Pearson Education, Inc. All rights reserved. This material is protected under all copyright laws as they currently exist.
No portion of this material may be reproduced, in any form or by any means, without permission in writing from the publisher.

Magnetism

20-19

The particle has an atomic mass of a multiple of 2.0 u. The simplest two cases are that it could be a
hydrogen-2 nucleus (called a deuteron) or a helium-4 nucleus (called an alpha particle):
60.

2
4
1 H, 2 He .

The particles in the mass spectrometer follow a semicircular path as shown in Fig. 2041. A particle
has a displacement of 2r from the point of entering the semicircular region to where it strikes the film.
So if the separation of the two molecules on the film is 0.50 mm, then the difference in radii of the two
molecules is 0.25 mm. The mass to radius ratio is the same for the two molecules.
q B = m 2 /r

m = qBr /

m
m
=
r CO r N 2

m /r = constant

28.0106 u
28.0134 u
=
r
r + 2.5 104 m

r = 2.5 m

The field inside the solenoid is given by Eq. 208 with 0 replaced by the permeability of the iron.

61.

B=

NI
A

3000(4 107 T m/A)(380)(0.35 A)


= 0.5 T
(1.0 m)

We assume that the factor of 3000 only has 1 significant figure.


The field inside the solenoid is given by Eq. 208 with 0 replaced by the permeability of the iron.

62.

B=

63.

NI
A

BA (2.2 T)(0.38 m)
=
= 2.239 105 T m/A 2.2 105 T m/A 180
NI
(780)(48 A)

The magnetic permeability is found from the two fields.


B

=
B0 0

B0 = 0 nI ; B = nI ;

= 0

B
B0

Here is a data table with the given values as well as the calculated values of .
For the graph, we have not plotted the last three data
points so that the structure for low fields is seen. It
would appear from the data that the value of is
asymptotically approaching 0 for large fields.

B0 (104 T)

70

-4

(10 T-m/A)

60
50
40
30
20
10
0
0

-4

B 0 (10 T)

10

12

14

0
0.13
0.25
0.50
0.63
0.78
1.0
1.3
1.9
2.5
6.3
13.0
130
1300
10000

B (T)
0
0.0042
0.01
0.028
0.043
0.095
0.45
0.67
1.01
1.18
1.44
1.58
1.72
2.26
3.15

(104 )(T-m/A)
4.06
5.03
7.04
8.58
15.3
56.5
64.8
66.8
59.3
28.7
15.3
1.66
0.218
0.040

Copyright 2014 Pearson Education, Inc. All rights reserved. This material is protected under all copyright laws as they currently exist.
No portion of this material may be reproduced, in any form or by any means, without permission in writing from the publisher.

20-20

64.

Chapter 20

(a)

Use Eq. 206 to calculate the field due to a long straight wire.
BA at B =

0 I A
(4 107 T m/A)(2.0 A)
=
= 2.667 106 T 2.7 106 T
2 rA to B
2 (0.15 m)

0 I B
(4 107 T m/A)(4.0 A)
=
= 5.333 106 T 5.3 106 T
2 rB to A
2 (0.15 m)

(b)

BB at A =

(c)

The two fields are not equal and opposite. Each individual field is due to a single wire and has no
dependence on the other wire. The magnitude of current in the second wire has nothing to do
with the value of the field caused by the first wire.
Use Eq. 207 to calculate the force per unit length on one wire due to the other wire. The forces
are attractive since the currents are in the same direction.

(d)

Fon A due to B Fon B due to A 0 I A I B


(4 107 T m/A) (2.0 A)(4.0 A)
=
=
=
AA
AB
2 d A to B
2
(0.15 m)
= 1.067 105 N/m 1.1 105 N/m

These two forces per unit length are equal and opposite because they are a Newtons third law
pair of forces.
65.

The magnetic force produces centripetal acceleration.


q B = m 2 /r m = p = qBr B =

4.8 1021 kg m/s


p
=
= 2.7 102 T
qr (1.60 1019 C)(1.1 m)

The magnetic field must point upward to cause an inward-pointing (centripetal) force that steers the
protons clockwise.
66.

There will be no force on either the top or bottom part of the wire, because the current is either parallel
to or opposite to the magnetic field. So the only force is on the left branch, which we define to be of
length A. Since the current is perpendicular to the magnetic field, use Eq. 202. The magnetic field can
be calculated by Eq. 208 for the magnetic field inside a solenoid. By the right-hand rule, the force on
the left branch is up out of the page.
(4 107 T m/A)(700)(7.0 A)
NI

F = I wire A wire B = I wire A wire 0 solenoid = (5.0 A)A wire

0.15 m

A solenoid
= 0.2A wire N (assuming A wire given in m)

67.

G
FB

We assume that the horizontal component of the


Earths magnetic field is pointing due north. The
Earths magnetic field also has the dip angle of 22.
The angle between the magnetic field and the eastward
current is 90. Use Eq. 201 to calculate the magnitude
of the force.
F = I AB sin = (330 A)(18 m)(5.0 105 T)sin 90

68o

22o
G
B Earth

= 0.297 N 0.30 N

Using the right-hand rule with the eastward current and the Earths magnetic field, the force on the
wire is northerly and 68 above the horizontal.

Copyright 2014 Pearson Education, Inc. All rights reserved. This material is protected under all copyright laws as they currently exist.
No portion of this material may be reproduced, in any form or by any means, without permission in writing from the publisher.

Magnetism

68.

From Example 206, we have r =

20-21

m
p
. The quantity m is the momentum, p, so r =
. Thus
qB
qB

p = qBr .

69.

The airplane is a charge moving in a magnetic field. Since it is flying perpendicular to the magnetic
field, Eq. 204 applies.
Fmax = q B = (1280 106 C)(120 m/s)(5.0 105 T) = 7.7 106 N

70.

The field inside a solenoid is given by Eq. 208.


B=

71.

0 NI

N=

BA
(0.050 T)(0.32 m)
=
= 1989 2.0 103 turns
0 I (4 107 T m/A)(6.4 A)

The magnetic force must be equal in magnitude to the weight of the electron.
mg = q B =

(9.11 1031 kg)(9.80 m/s 2 )


mg
=
= 1.1 106 m/s
qB (1.60 1019 C)(0.50 104 T)

The magnetic force must point upward, so by the right-hand rule and the negative charge of the
electron, the electron must be moving west.
72.

(a)

The velocity of the ions is found using energy conservation. The electrical potential energy of
the ions becomes kinetic energy as they are accelerated. Then, assuming the ions move
perpendicularly to the magnetic field, the magnetic force will be a maximum. That force will
cause the ion to move in a circular path.
PE initial

= KE final

m
r=
=
qB

(b)

2qV
m =
qB

2mV
qB 2

2qV
m

q B =

2(6.6 1027 kg)(3200 V)


2(1.60 1019 C)(0.240 T) 2

m 2
r

= 4.787 102 m 4.8 102 m

The period is the circumference of the circular path divided by the speed.

T=

73.

qV = 12 m 2

2 r

2
=

2mV
qB 2
2qV
m

2 m
2 (6.6 1027 kg)
=
= 5.5 107 s
qB
2(1.60 1019 C)(0.240 T)

The magnetic field at the center of the square is the vector sum of the
magnetic field created by each current. Since the magnitudes of the
currents are equal and the distance from each corner to the center is the
same, the magnitude of the magnetic field from each wire is the same
and is given by Eq. 206. The direction of the magnetic field is directed
by the right-hand rule and is shown in the diagram. By symmetry, we
see that the vertical components of the magnetic field cancel and the
horizontal components add.

A
Copyright 2014 Pearson Education, Inc. All rights reserved. This material is protected under all copyright laws as they currently exist.
No portion of this material may be reproduced, in any form or by any means, without permission in writing from the publisher.

20-22

Chapter 20

G G
G
G
G
I
B = B1 + B 2 + B3 + B 4 = 4 0 cos 45 to the left
2 r

I 2
2 0 I
0

= 4
to the left =
to the left
A

2 2
A
2

74.

(a)

For the beam of electrons to be undeflected, the magnitude of the magnetic force must equal the
magnitude of the electric force. We assume that the magnetic field will be perpendicular to the
velocity of the electrons so that the maximum magnetic force is obtained.
FB = FE

12,000 V/m
6

4.8 10 m/s

= 2.5 103 T

Since the electric field is pointing south, the electric force is to the north. Thus the magnetic
force must be to the south. Using the right-hand rule with the negative electrons, the magnetic
field must be vertically upward.

(c)

If the electric field is turned off, then the magnetic field will cause a centripetal force, moving
the electrons in a circular path. The frequency is the cyclotron frequency, Eq. 205.
(1.60 1019 C)(12,000 V/m)
qB
qE
=
=
= 7.0 107 Hz
2 m 2 m 2 (9.11 1031 kg)(4.8 106 m/s)

The protons will follow a circular path as they move through the
region of magnetic field, with a radius of curvature given in
m
Example 206 as r =
. Fast-moving protons will have a radius
qB
of curvature that is too large, so they will exit above the second
tube. Likewise, slow-moving protons will have a radius of curvature
that is too small, so they will exit below the second tube. Since the
exit velocity is perpendicular to the radius line from the center of
curvature, the bending angle can be calculated.
sin =

A
r
A
r

0 I
2r

AqB
(5.0 102 m)(1.60 1019 C)(0.41 T)
= sin 1
= sin 1 0.7856 = 52
mv
(1.67 1027 kg)(2.5 106 m/s)

With one turn of wire, we have B =

B=

= sin 1 = sin 1

76.

(b)

f =

75.

q B = qE B =

I=

2rB

0 I
2r

. Use the radius of the Earth for r.

2(6.38 106 m)(1 104 T)


4 107 T m/A

= 1.015 109 A 1 109 A

Copyright 2014 Pearson Education, Inc. All rights reserved. This material is protected under all copyright laws as they currently exist.
No portion of this material may be reproduced, in any form or by any means, without permission in writing from the publisher.

Magnetism

77.

20-23

The speed of the proton can be calculated based on the radius of curvature of the (almost) circular
motion. From that the kinetic energy can be calculated.
2

q B =

m 2
r

KE =

q2 B2 2 2
(1.60 1019 C) 2 (0.010 T) 2
(r2 r1 ) =
[(8.5 103 m) 2 (10.0 103 m) 2 ]
27
2m
2(1.67 10 kg)

qBr
m

q2 B2r 2

=
2m
m

qBr
2
KE = 1 m = 1 m
2

= 2.1 1020 J or 0.13 eV

78.

There are three forces on each wireits weight, the magnetic force of repulsion
from the other wire, and the tension in the attached string. See the diagram. The
magnetic force is given by Eq. 207. The mass of the wire is its density times its
volume. The length of the current-carrying wires is A. The net force in both the
vertical and horizontal directions is zero.
FB = FT sin ; FT cos = mg

FT =

mg
cos

FB =

mg
2
sin = mg tan = Al rAl
Ag tan
cos

FB =

0 I 2
0 I 2
2
A
A = Al rAl
Ag tan
2 S
2 S

I=

2
g tan
2 S Al rAl

G
FB

G
F
T

G
mg

2 [2(0.50 m) sin 3](2700 kg/m3 ) (2.1 104 m) 2 (9.8 m/s 2 ) tan 3

(4 107 T m/A)

= 7.1 A

79.

The centripetal force is caused by the magnetic field and is given by Eq. 203.
F = q B sin = q B = m
r=

2
r

m (9.11 1031 kg)(3.0 106 m/s) sin 45


=
= 5.251 105 m 5.3 105 m
19
qB
(1.60 10
C)(0.23 T)

The component of the velocity that is parallel to the magnetic field is unchanged, so the pitch is that
velocity component times the period of the circular motion.

T=

2 r

2
=

m
qB

2 m
qB

2 m
2 (9.11 1031 kg)
6
= 3.3 104 m
p = &T = cos 45
= (3.0 10 m/s) cos 45
19
qB
(1.60 10
C)(0.23 T)

80.

The maximum torque is found using Eq. 2010 with sin = 1. Set the current equal to the voltage
divided by the resistance and the area as the square of the side length.
9.0 V
V 2
2
4
(0.050 m) (0.020 T) = 3.2 10 m N
A B = 20
R
28

= NIAB = N

Copyright 2014 Pearson Education, Inc. All rights reserved. This material is protected under all copyright laws as they currently exist.
No portion of this material may be reproduced, in any form or by any means, without permission in writing from the publisher.

20-24

81.

Chapter 20

We find the speed of the electron using conservation of


energy. The accelerating potential energy becomes the
kinetic energy of the electron.
eV = 12 m 2

2eV
=
m

2(2200 eV)(1.60 1019 J/eV)


(9.11 1031 kg)

= 2.78 107 m/s

Upon entering the magnetic field, the electron is


traveling horizontally. The magnetic field will cause the
path of the electron to be an arc of a circle of radius r
and deflect an angle from the horizontal. While in the field, the electron will travel a horizontal
distance d and a vertical distance h0 . Our approximation will be to ignore the distance h0 . We have
the following relationships.
h
d
m
; sin = ; r =

A
r
eB
h
11 cm
d
0.038 m
= tan 1 = tan 1
= 26.6; r =
=
= 0.0849 m
A
22 cm
sin sin 26.6
tan =

B=

82.

m (9.11 1031 kg)(2.78 107 m/s)


=
= 1.86 103 T 1.9 103 T
er
(1.60 1019 C)(0.0849 m)

The magnetic field from the wire at the location of the plane is perpendicular to the velocity of the
plane since the plane is flying parallel to the wire. We calculate the force on the plane, and thus the
acceleration, using Eq. 204, with the magnetic field of the wire given by Eq. 206.
F = q B = q
a=

0 I
2 r

F q 0 I (18.0 103 C)(3.4 m/s)(4 107 T m/A)(25 A)


=
=
m m 2 r
2 (0.175 kg)(0.086 m)

1g
= 2.033 105 m/s 2
9.80 m/s 2

83.

6
= 2.1 10 g's

Since the magnetic and gravitational force along the entire rod is uniform, we consider the two forces
acting at the center of mass of the rod. To be balanced, the net torque about the fulcrum must be zero.
Using the usual sign convention for torques and Eq. 810a, we solve for the magnetic force on the rod.

= 0 = Mg ( 14 A ) mg ( 14 A ) FM ( 14 A )

FM = (M m)g

We solve for the current using Eq. 202.


I=

F (M m)g (6.0m m)g 5.0mg


=
=
=
AB
AB
AB
AB

The right-hand rule indicates that the current must flow toward the left since the magnetic field is into
the page and the magnetic force is downward.

Copyright 2014 Pearson Education, Inc. All rights reserved. This material is protected under all copyright laws as they currently exist.
No portion of this material may be reproduced, in any form or by any means, without permission in writing from the publisher.

Magnetism

84.

20-25

The magnetic force will produce centripetal acceleration. Use that relationship to calculate the speed.
The radius of the Earth is 6.38 106 km, and the altitude is added to that.
FB = q B = m

2
r

qrB (1.60 1019 C)(6.385 106 m)(0.50 104 T)


=
= 1.3 108 m/s
27
m
238(1.66 10
kg)

Compare the size of the magnetic force to the force of gravity on the ion.
FB q B (1.60 1019 C)(1.3 108 m/s)(0.50 104 T)
=
=
= 2.7 108
Fg
mg
238(1.66 1027 kg)(9.80 m/s 2 )
It is fine to ignore gravitythe magnetic force is almost 300 million times larger than gravity.
85.

(a)

For the particle to move upward, the magnetic force must point upward; by the right-hand rule
we see that the force on a positively charged particle would be downward. Therefore, the charge
on the particle must be negative.

(b)

In the figure we have created a right triangle to relate the


horizontal distance A, the displacement d, and the radius of
curvature, r. Using the Pythagorean theorem we can write
an expression for the radius in terms of the other two
distances.
r 2 = (r d ) 2 + A 2

r=

d 2 + A2
2d

Since the momentum is perpendicular to the magnetic field,


we can solve for the momentum by relating the maximum
force (Eq. 204) to the centripetal force on the particle.
Fmax = q B0 =

86.

m 2
r

p = m = qB0 r =

qB0 (d 2 + A 2 )
2d

Example 1810 estimates the current in a lightning bolt at 100 A. Use Eq. 202, with the Earths
magnetic field. We estimate the flag pole as being 65 m tall.
F = I AB = (100 A)(6 m)(0.5 104 T) = 0.03 N

87.

The accelerating force on the bar is due to the magnetic force on the current. If the current is constant,
then the magnetic force will be constant, so constant-acceleration kinematics (Eq. 211c) can be used.

2 = 02 + 2ax a =

Fnet

2 02

2x
2x
2
m
2x
2
2
3
ma
= mv = (1.5 10 kg)(28 m/s) = 1.2 A
= ma = I AB I =
=
AB
AB
2ABx 2(0.28 m)(1.7 T)(1.0 m)

Using the right-hand rule, for the force on the bar to be in the direction of the acceleration shown in
Fig. 2071, the magnetic field must be down.

Copyright 2014 Pearson Education, Inc. All rights reserved. This material is protected under all copyright laws as they currently exist.
No portion of this material may be reproduced, in any form or by any means, without permission in writing from the publisher.

20-26

88.

Chapter 20

(a)

The frequency of the voltage must match the frequency of circular motion of the particles so that
the electric field is synchronized with the circular motion. The radius of each circular orbit is
m
. For an object moving in circular motion, the period is given
given in Example 206 as r =
qB
by T =
T=

(b)

2 r

, and the frequency is the reciprocal of the period.

2 r

f =

Bq
=
=
T 2 m
2 m
qB

In particular, we note that this frequency is independent of the radius, so the same frequency can
be used throughout the acceleration. This frequency is also given as Eq. 205.
For a small gap, the electric field across the gap will be approximately constant and uniform as
the particles cross the gap. If the motion and the voltage are synchronized so that the maximum
voltage occurs when the particles are at the gap, then the particles receive an energy increase of
KE = qV0 as they pass each gap. The energy gain from one revolution will include the passing of
two gaps, so the total KE increase is 2qV0 .

(c)

The maximum kinetic energy will occur at the outside of the cyclotron.

rqB
m
2

KE max

r qB
2
= 12 mmax
= 12 m max =
m

1 eV
= 7.66 1012 J
1.60 1019

89.

(a)

2
2 2
1 rmax q B
2
m

1
2

(2.0 m) 2 (1.60 1019 C) 2 (0.50 T) 2


1.67 1027 kg

1 MeV

= 48 MeV
6
J
10 eV

The forces on wire M due to the other wires are repelling forces, away
from the other wires. Use Eq. 207 to calculate the force per unit length
on wire M due to each of the other wires and then add the force vectors
together. The horizontal parts of the forces cancel, and the sum is
vertical.
FM net y
AM
=

G
FMP
30o 30o

G
FMN

FMN
F
cos 30 + MP cos 30
AM
AM

0 I M I N
I I
cos 30 + 0 M P cos 30
2 d MN
2 d MP

=2

(4 107 T m/A) (8.00 A) 2


cos 30 = 5.8 104 N/m, upward
2
(0.038 m)

The forces on wire N due to the other wires are an attractive force
toward wire P and a repelling force away from wire M. Use Eq. 207
to calculate the force per unit length on wire N due to each of the other
wires and then add the force vectors together. From symmetry, we
expect the net force to lie exactly between the two individual force
vectors, which is 60 below the horizontal.

G
FNP
G
FNM

90o
o

30

Copyright 2014 Pearson Education, Inc. All rights reserved. This material is protected under all copyright laws as they currently exist.
No portion of this material may be reproduced, in any form or by any means, without permission in writing from the publisher.

Magnetism

FN net y = FNM cos 30 =


=

20-27

0 I M I N
cos 30
2 d MN

(4 107 T m/A) (8.00 A) 2


cos 30 = 2.917 104 N/m
2
(0.038 m)

FN net x = FNM sin 30 + FNP


=

(4 107 T m/A) (8.00 A) 2


(4 107 T m/A) (8.00 A) 2
sin 30 +
2
(0.038 m)
2
(0.038 m)

= 1.684 105 N/m


FN net = FN2 net x + FN2 net y = 3.4 104 N/m = tan 1

FN net y
FN net x

= tan 1

2.917
= 300
1.684

The forces on wire P due to the other wires are an attractive force toward
wire N and a repelling force away from wire M. Use Eq. 207 to calculate
the force per unit length on wire P due to each of the other wires and then
add the force vectors together. From symmetry, this is just a mirror image
of the previous solution, so the net force is as follows.
FP net = 3.4 104 N/m
(b)

30o

G
BP

G
G
G
G
B net = B M + B N + B P

BP =

G
FPM

90o

= 240

There will be three magnetic fields to sumone from each wire. Each
field will point perpendicularly to the line connecting the wire to the
midpoint. The two fields due to M and N are drawn slightly separated
from each other, but should be collinear. The magnitude of each field
is given by Eq. 206.

BM = BN =

G
FPN

0 I
(4 107 m/A) 8.00 A
=
= 8.421 105 T
2 rM
2
0.019 m

0 I (4 107 T m/A) 8.00 A


=
= 4.862 105 T
2 rP
2
3(0.019 m)

60o
30o G
BM
G
BN
P

Bnet x = 2 BM cos 30 + BP cos 60 = 2(8.421 105 T) cos 30


+ (4.862 105 T)cos 60 = 1.702 104 T
Bnet y = 2 BM sin 30 + BP sin 60 = 2(8.421 105 T) sin 30
+ (4.862 105 T)sin 60 = 4.210 105 T
2
2
4
Bnet = Bnet
T) 2 + ( 4.210 105 T) = 1.75 104 T
x + Bnet y = (1.702 10

net = tan 1

Bnet y
Bnet x

= tan 1

4.210 105 T
1.702 104 T

= 14

The net field points slightly below the horizontal direction.


90.

Each of the bottom wires will repel the top wire since each bottom current is
opposite to the top current. The repelling forces will be along the line directly from
the bottom wires to the top wires. Only the vertical components of those forces will
be counteracting gravity. Use Eq. 207 to calculate the magnetic forces. The mass of
the wire is its density times its volume. The length of the wire is represented by A.

G
FB

30o 30o

G
FB

G
mg

Copyright 2014 Pearson Education, Inc. All rights reserved. This material is protected under all copyright laws as they currently exist.
No portion of this material may be reproduced, in any form or by any means, without permission in writing from the publisher.

20-28

Chapter 20

mg = 2 FB cos 30 Cu A

Cu 2 ( 12 d Cu ) dg
2

I Cu =

0 I bottom cos 30

( 12 dCu )

g=2

0 I bottom I Cu

A cos 30

(8900 kg/m3 ) 2 (5.0 104 m) 2 (0.038 m)(9.80 m/s 2 )


(4 107 T m/A)(75 A)( cos 30)

= 100.2 A 1.0 102 A

91.

We approximate the magnetic field by using Eq. 206. The current is found from Eq. 185.
I=

P
V

B=

0 I
P (4 107 T m/A) (46 106 W)
= 0
=
= 2.949 106 T 2.9 106 T
5
2 r 2 r V
2 (13 m)
(2.4 10 V)

Bwire 2.949 106 T


=
= 5.90 102 6%
BEarth
5 105 T

The power line is almost certainly AC, so the voltage, power, current, and the magnitude of the
magnetic field are most likely rms values.
92.

The net magnetic field is the vector sum of the magnetic fields
produced by each current-carrying wire. Since the individual
magnetic fields encircle the wire producing it, the field is
perpendicular to the radial line from the wire to point P. We
G
G
let B1 be the field from the left wire and B 2 designate the
field from the right wire. The magnitude of the magnetic field
vectors is calculated from Eq. 206.
B1 =

0 I (4 107 T m/A)(19.2 A)
=
= 3.2000 105 T
2 r1
2 (0.12 m)

B2 =

0 I (4 107 T m/A)(19.2 A)
=
= 2.9538 105 T
2 r2
2 (0.13 m)

We use the law of cosines to determine the angle that the radial line from each wire to point P makes
with the horizontal. Since the magnetic fields are perpendicular to the radial lines, these angles are the
same as the angles the magnetic fields make with the vertical.
(0.12 m) 2 + (0.082 m) 2 (0.13 m) 2
= 77.606

2(0.12 m)(0.082 m)

2
2
2

(0.13 m) + (0.082 m) (0.12 m)


2 = cos 1
= 64.364

2(0.13 m)(0.082 m)

1 = cos 1

Using the magnitudes and angles of each magnetic field, we calculate the horizontal and vertical
components, add the vectors, and calculate the resultant magnetic field and angle.
Bnet x = B1 sin (1 ) B2 sin 2 = (3.2000 105 T)sin 77.606 (2.9538 105 T) sin 64.364
= 57.88 106 T
Bnet y = B1 cos (1 ) B2 cos 1 = (3.2000 105 T) cos 77.606 (2.9538 105 T) cos 64.364
= 5.911 106 T
Copyright 2014 Pearson Education, Inc. All rights reserved. This material is protected under all copyright laws as they currently exist.
No portion of this material may be reproduced, in any form or by any means, without permission in writing from the publisher.

Magnetism

20-29

2
2
6
B = Bnet
T)2 + (5.911 106 T) 2 = 5.82 105 T
x + Bnet y = (57.88 10

= tan 1

Bnet y
Bnet x

= tan 1

5.911 106 T
57.88 106 T

= 5.83

G
B = 5.82 105 T at 5.83 below the negative x axis

Solutions to Search and Learn Problems


1.

There are three magnetic force equations given in this chapter.


(i)

(ii)

F = I AB sin

F = q B sin

This is the force on a current-carrying wire in a magnetic field. The current is


measured in amperes, the length in meters, and the magnetic field in teslas.
A tesla is equivalent to a newton per ampere per meter.
N
(A)(m)(T) = (A)(m)
= (N)
Am
This is the force on a moving charged particle in a magnetic field. The charge is
measured in coulombs, the velocity in meters per second, and the magnetic field
in teslas.

m
C
N
(C) (T) = (m)(T) = (A)(m)
= (N)
s
s


Am

(iii)

F=

0 I1 I 2
A2
2 d

This is the force between two parallel currentsspecifically, the force on a


length A 2 of the wire carrying current I 2 , due to another wire carrying current
I1. The permeability of free space has units of T m/A.
T m (A)(A)
N

(m) = (T m)(A) =
m (A) = (N)

A (m)
Am

2.

(a)
(b)
(c)

The electron, a negative particle, accelerates opposite the direction of the electric field.
Therefore, for the electron to accelerate east, the electric field should point to the west.
By the right-hand rule, the magnetic field must point upward for the magnetic force on a northmoving electron to point westward.
For the electron to not accelerate, the magnetic force must be equal in magnitude but opposite in
direction to the electric force. Set the magnitudes of the forces from Eq. 165 and Eq. 204 equal
and solve for the magnetic field. The direction of the field is upward.

qE = q B B =
(d)

(e)

330 V/m
3.0 104 m/s

= 0.011 T

If the electron is moving faster, then the magnetic force will be greater than the electric force and
the electron will accelerate westward. If it is moving slower, then the magnetic force will be
smaller than the electric force and the electron will accelerate eastward.
The two forces must be equal and opposite for the particle to travel undeflected. The ratio of the
electric to magnetic field can be calculated by setting the forces equal.
E
= = 5.5 104 m/s
B
The value of B cannot be determined unless the value of E is known.
qE = q B

Copyright 2014 Pearson Education, Inc. All rights reserved. This material is protected under all copyright laws as they currently exist.
No portion of this material may be reproduced, in any form or by any means, without permission in writing from the publisher.

20-30

3.

Chapter 20

(a)

In Example 206, the radius of curvature for an electron in a uniform magnetic field was shown
to be r = m /qB. If the kinetic energy (and therefore the speed) of the particle remains constant
as the magnetic field doubles, then the radius of curvature is cut in half. The angular momentum
of a particle is given by L = m r , so if the speed remains constant as the radius of curvature
decreases, the angular momentum will be cut in half.

(b)

The magnetic dipole moment is defined in Eq. 2011 as M = NIA. The number of turns, N, is 1.
The current is the charge per unit time passing a given point, which on the average is the charge
on the electron divided by the period of the circular motion, I = e /T . If we assume that the
electron is moving in a circular orbit of radius r, then the area is A = r 2 . The period of the
motion is the circumference of the orbit divided by the speed, T = 2 r / . Finally, the angular
momentum of an object moving in a circle is given by L = m r. Combining these relationships
gives the magnetic moment in terms of the angular momentum.
M = NIA =

4.

(a)

e 2
e
e r 2 er em r
e
e
=
=
=
m r =
L
r =
r2 =
T
2 r /
2 r
2
2m
2m
2m

The left current creates a field on the x axis that points in the positive y direction, and the right
current creates a field on the x axis pointing in the negative y direction. If the distance from the
left wire to a point on the positive x axis is x, then the distance from the right wire to that same
point is d x. We can write the net magnetic field, taking upward as positive, using Eq. 206.
Bnet =

0 I
0 I
I 1
1 0 I d 2 x

= 0

=
2 x 2 (d x) 2 x d x
2 x(d x)

(b)

5.

Using the configuration shown in Fig. 2076, the downward electric field puts a force on the charged
particles. Positive charges are accelerated downward, and negative particles are accelerated upward.
As the ions move vertically, perpendicularly to the magnetic field, the magnetic field now puts a force
on them. Using right-hand rule 1, we see that both charges of ions experience a force out of the page,
which is parallel to the axis of the blood vessel.

Copyright 2014 Pearson Education, Inc. All rights reserved. This material is protected under all copyright laws as they currently exist.
No portion of this material may be reproduced, in any form or by any means, without permission in writing from the publisher.

Das könnte Ihnen auch gefallen